返回列表 发帖

求助 og048

48. A researcher discovered that people who have low levels of immune-system activity tend to score much lower on tests of mental health than do people with normal or high immune-system activity. The researcher concluded from this experiment that the immune system protects against mental illness as well as against physical disease

The researcher’s conclusion depends on which of the following assumptions?

A. High immune-system activity protects against mental illness better than normal immune-system activity does.

B. Mental illness is similar to physical disease in its effects on body systems.

C. People with high immune-system activity cannot develop mental illness.

D. Mental illness does not cause people’s immune-system activity to decrease. D

E. Psychological treatment of mental illness is not as effective as is medical treatment.

48.The researcher concludes from the association of low immune-system activity with low mental-health sores that, in effect, immune system activity can inhibit mental illness. If, contrary to D, mental illness can depress immune-system activity, the association mentioned does not support the researcher’s conclusion. So D must be assumed.

这里是在取非么,对d选项取非后是心理疾病会引起免疫系统下降,和原文有什么矛盾的呢?

收藏 分享

thanks a lot !!!!!!!

TOP

我的理解如下:

这是个因果型结论,OG的解释认为low levels of immune-system activity和lower test score of mental health同时存在,所以认为结论这两者有因果关系,而且是前者引起后者。作为assumption题的答案D,应该是否认因果倒置的可能性,也就是说不是后者引起前者。如果将这个答案取非,的确是后者引起前者,题目中的结论就是错误的。

TOP

返回列表

站长推荐 关闭


美国top10 MBA VIP申请服务

自2003年开始提供 MBA 申请服务以来,保持着90% 以上的成功率,其中Top10 MBA服务成功率更是高达95%


查看